【統計検定1級過去問】2015年(統計数理)大問1 解答例

投稿者: | 2017-08-14

コメント

大問1はいかにも数理統計な問題が出題されました。標本平均、不偏分散の平均、分散およびモーメント算出、推定量の不偏性、一致性と盛りだくさんな内容です。

問3が方針次第でやや煩雑になるのと問4でチェビシェフの不等式に気づくかがポイントで、統計検定にしては珍しく最後の問5が問3, 4と比べると簡単です。

基本に忠実に解いていけば完答も難しくない問題セットです。また、各小問はほぼ独立した問題になっており仮に分からない問題があってもあきらめずに次の問題に着手して解ける問題で確実に点が取れたかが勝負の分かれ目になっているのではと思います。

問題

平均[math]\mu[/math], 分散[math]\sigma^2[/math]をパラメータとして持つ分布を分布[math]D[/math]とする。[math]X_1,\dots,X_n \sim D[/math]を互いに独立な確率変数とし、[math]\bar{X}=\frac{1}{n}\sum_{i=1}^nX_i[/math], [math]S^2=\frac{1}{n-1}\sum_{i=1}^n(X_i-\bar{X})^2[/math]とする時、以下の問に答えよ。

(出典:「統計検定 1級・準1級 公式問題集」。問題文を一部略記。)

問1

[math]E\left[\bar{X}^2\right][/math]を求めよ。

[math]\bar{X}[/math]の平均、分散を求めると

[math]
\begin{eqnarray}
E\left[\bar{X}\right] &=& E\left[\frac{1}{n}\sum_{i=1}^nX_i\right] \\
&=& \frac{1}{n}\sum_{i=1}^nE[X_i] \\
&=& \mu
\end{eqnarray}
[/math]

および

[math]
\begin{eqnarray}
V[\bar{X}] &=& V\left[\frac{1}{n}\sum_{i=1}^nX_i\right] \\
&=& \frac{1}{n^2}\sum_{i=1}^nV[X_i] \\
&=& \frac{\sigma^2}{n}
\end{eqnarray}
[/math]

なので

[math]
\begin{eqnarray}
E\left[\bar{X}^2\right]&=&V[\bar{X}]+\left(E\left[\bar{X}\right]\right)^2 \\
&=&\frac{\sigma^2}{n}+\mu^2
\end{eqnarray}
[/math]

である。

問2

[math]S^2[/math]は[math]\sigma^2[/math]の不偏推定量であることを示せ。

偏差平方和[math](n-1)S^2=\sum_{i=1}^n (X_i-\bar{X})^2[/math]を平均[math]\mu[/math]周りで展開すると

[math]
\begin{eqnarray}
&& \sum_{i=1}^n (X_i-\bar{X})^2 \\
&=& \sum_{i=1}^n \left(X_i-\mu -(\bar{X}-\mu)\right)^2 \\
&=& \sum_{i=1}^n \left\{(X_i-\mu)^2\right. \\
&&\left.-2(X_i-\mu)(\bar{X}-\mu) + (\bar{X}-\mu)^2\right\}
\end{eqnarray}
[/math]

であり、第2項は

[math]
\begin{eqnarray}
&&\sum_{i=1}^n(X_i-\mu)(\bar{X}-\mu)\\
&=&(\bar{X}-\mu)\sum_{i=1}^n(X_i-\mu) \\
&=& n(\bar{X}-\mu)^2
\end{eqnarray}
[/math]

なので

[math]
(n-1)S^2=\sum_{i=1}^n (X_i-\mu)^2-n(\bar{X}-\mu)^2
[/math]

である。これより

[math]
\begin{eqnarray}
&&E[(n-1)S^2] \\
&=& \sum_{i=1}^n E\left[(X_i-\mu)^2\right]-nE\left[(\bar{X}-\mu)^2\right] \\
&=& n \sigma^2 – n \cdot \frac{\sigma^2}{n} \\
&=& (n-1)\sigma^2
\end{eqnarray}
[/math]

なので

[math]
E[S^2]=\sigma^2
[/math]

が成立し、[math]S^2[/math]は[math]\sigma^2[/math]の不偏推定量である。

問3

以下では分布[math]D[/math]は正規分布[math]\mathcal{N}(\mu,\ \sigma^2)[/math]とする。[math]k\in\mathbb{N}[/math]に対して[math]E\left[\bar{X}^k\right][/math]を求めよ。
まずは[math]\bar{X}\sim\mathcal{N}(0, \frac{\sigma^2}{n})[/math]なので標準正規分布[math]\mathcal{N}(0,\ 1)[/math]に従う確率変数[math]Z[/math]を用いて[math]\bar{X}=\frac{\sigma}{\sqrt{n}}Z[/math]と書けるので標準正規分布の[math]k[/math]次モーメント[math]E[Z^k][/math]を求めれば良いことに気づきましょう。

ここから方針は大きく2つあります。

  • [math]E[Z^k][/math]をガンマ分布の積分に帰着し計算する。
  • モーメント母関数[math]M_Z(t)[/math]をTaylor展開して[math]k[/math]次微分係数を求める。

ここでは「モーメント母関数によるモーメント算出」で紹介したTaylor展開を使った解法で示します。

[math]\bar{X}\sim\mathcal{N}(0, \frac{\sigma^2}{n})[/math]であり[math]Z\sim\mathcal{N}(0,\ 1)[/math]を用いて[math]\bar{X}=\frac{\sigma}{\sqrt{n}}Z[/math]とかけ

[math]
E\left[\bar{X}^k\right]=\left(\dfrac{\sigma^2}{n}\right)^{{k/2}}E[Z^k]
[/math]

となるので[math]Z[/math]の[math]k[/math]次モーメントを求める。

[math]Z[/math]のモーメント母関数は[math]M_Z(t)=e^{t^2/2}[/math]であり、Taylor展開をして

[math]
\begin{eqnarray}
M_Z(t) &=& e^{t^2/2} \\
&=& \sum_{l=0}^{\infty} \dfrac{1}{l!}\left(\frac{t^2}{2}\right)^l \\
&=& \sum_{l=0}^{\infty} \dfrac{(2l)!}{2^l l!}\cdot \dfrac{t^{2l}}{(2l)!}
\end{eqnarray}
[/math]

とかけるので[math]k[/math]次微分係数[math]c_k(=E[Z^k])[/math]は[math]l\in\mathbb{N}[/math]を用いて

[math]
c_k = \begin{cases}
0 &(k=2l-1) \\
\dfrac{(2l)!}{2^l l!} &(k=2l)
\end{cases}
[/math]

となる。以上より

[math]
E\left[\bar{X}^k\right]=\begin{cases}
0 &(k=2l-1) \\
\dfrac{(2l)!}{l!}\left(\dfrac{\sigma^2}{2n}\right)^l &(k=2l)
\end{cases}
[/math]

を得る。

問4

[math]S^2[/math]は[math]\sigma^2[/math]の一致推定量かどうか述べよ。

任意の[math]\epsilon>0[/math]に対してチェビシェフ不等式より

[math]
\begin{eqnarray}
P\left( \left(S^2-\sigma^2 \right)^2 > \epsilon \right) &\leq& \dfrac{E\left[\left(S^2-\sigma^2 \right)^2\right]}{\epsilon} \\
&=& \dfrac{V\left[S^2\right]}{\epsilon}
\end{eqnarray}
[/math]

が成立する。

ここで[math](n-1)S^2/\sigma^2\sim \chi^2(n-1)[/math]なので[math]V\left[(n-1)S^2/\sigma^2 \right]=2(n-1)[/math]より

[math]
V\left[S^2\right]=\dfrac{2\sigma^4}{(n-1)}
[/math]

である。これより

[math]
\begin{eqnarray}
P\left( \left(S^2-\sigma^2 \right)^2 > \epsilon \right) &\leq& \dfrac{V\left[S^2\right]}{\epsilon} \\
&=& \dfrac{2\sigma^4}{(n-1)\epsilon}
\end{eqnarray}
[/math]

であり、[math]P\left( \left(S^2-\sigma^2 \right)^2 > \epsilon \right)\to 0\ (n\to\infty)[/math]が成立するので[math]S^2[/math]は[math]\sigma^2[/math]の一致推定量である。

問5

[math]c>0[/math]としたとき[math]MSE[cS^2]=E\left[(cS^2-\sigma^2)^2\right][/math]を求め、その値を最小にする[math]c[/math]と最小値を求めよ。

[math]cS^2[/math]の平均[math]c\sigma^2[/math]周りで展開すると

[math]
\begin{eqnarray}
&& (cS^2-\sigma^2)^2 \\
&=& (cS^2 – c\sigma^2 – (\sigma^2-c\sigma^2))^2 \\
&=& c^2(S^2 – \sigma^2)^2 \\
&&\ -2(1-c)\sigma^2(S^2 – \sigma^2)+(1-c)^2\sigma^4
\end{eqnarray}
[/math]

となり第2項は平均を取ると[math]0[/math]になるので

[math]
\begin{eqnarray}
&&MSE[cS^2]\\
&=&E\left[(cS^2-\sigma^2)^2\right]\\
&=&c^2V[S^2]+(1-c)^2\sigma^4 \\
&=&\left\{\dfrac{2c^2}{(n-1)}+(1-c)^2\right\}\sigma^4
\end{eqnarray}
[/math]

である。[math]\sigma^4[/math]の係数を[math]f(c)[/math]とおいて[math]f'(c)=0[/math]を解くと[math]c=\frac{n-1}{n+1}[/math]の時に最小になりそのときの[math]MSE[cS^2][/math]は[math]\frac{2\sigma^4}{n+1}[/math]である。

シリーズ記事

スポンサーリンク


コメントを残す

メールアドレスが公開されることはありません。 が付いている欄は必須項目です